Monday 26 October 2015

Tricky Cardiology Multiple Choice Questions And Answers

41- Which of the following compounds has a vasodilating effect?
1) Antidiuretic hormone
2) Calcitonin
3) Endothelin
4) Renin
5) Somatostatin
Answer-2

42- Which of the following may be responsible for a hypokalaemic hypertension ?
1) Non-classical congenital adrenal hyperplasia
2) Barter's syndrome
3) Diabetic nephropathy
4) Liddle's syndrome
5) Type IV renal tubular acidosis
Answer-4

43- A 52 year old sales representative is admitted with an inferior myocardial infarction. He receives thrombolysis and makes an uneventful recovery. He is discharged on atenolol, aspirin and  orvastatin. He enquires how long after his MI must he wait before he is able to drive?
1) One week
2) Two weeks
3) Four weeks
4) Three months
5) Six months
Answer-3

44- A 35 year old woman presented with a history of intermittent lightheadedness. Clinical examination and 12-lead ECG were normal. Which of the following, if present on a 24 hour Holter ECG tracing, would be the most clinically important?
1) Atrial premature beats.
2) Profound sleep-associated bradycardia.
3) Supraventricular tachycardia.
4) Transient Mobitz type 1 atrioventricular block.
5) Vertricular premature beats.
Answer-3

45- A 57-year-old man develops deep venous thrombosis during a hospitalization for prostatectomy. He exhibits decreased mental status with right hemiplegia, and a CT scan of the head suggests an acute cerebral infarction in the distribution of the left middle cerebral artery. A chest X-ray reveals cardiac enlargement and prominence of the main pulmonary arteries that suggests pulmonary hypertension. His serum troponin I is <0.4 ng/mL. Which of the following lesions is most likely to
be present on echocardiography?
1) Coarctation of the aorta
2) Dextrocardia
3) Pulmonary stenosis
4) Tetralogy of Fallot
5) Ventricular septal defect
Answer-5

46- A 60 year old man had a myocardial infarction 6 weeks ago. He is taking aspirin 75 mg/day and metoprolol 50mg 2/day. During a routine follow-up Exercise Test he has a 20 beat run of non-sustained VT. He achieved stage 4 of the Bruce protocol and 92 % of his target heart rate. The non-sustained VT occurred halfway through Stage 2. ST segments were normal during the study. What is the definitive investigation?
1) Coronary angiography.
2) Echocardiogram.
3) Electrophysiological study.
4) Thallium exercise scan.
5) 24 hour Holter monitor.
Answer-3

47- A 55-year-old woman has had worsening shortness of breath for several years. She now has to sleep sitting up on two pillows. She has difficulty swallowing. There is no history of chest pain. She is afebrile. Recently, she suffered a stroke with left hemiparesis. A chest X-ray reveals a near-normal left ventricular size with a prominent left atrial border. Which of the following conditions is most likely to account for these findings?
1) Aortic coarctation
2) Cardiomyopathy
3) Essential hypertension
4) Left renal artery stenosis
5) Mitral valve stenosis
Answer-5

48- Which of the following antiarrhythmic drugs may be used in the treatment of long QT syndrome?
1) Amiodarone
2) Atenolol
3) Flecainide
4) Propofanone
5) Sotalol
Answer-2

49- A 70 year old male was receiving amiodarone 200 mg daily for intermittent atrial fibrillation. However, he was aware of tiredness and lethargy. He appeared clinically euthyroid with no palpable goitre. Investigations revealed: Serum free T4 23pmol/L (9-26) Serum total T3 0.8 nmol/L (0.9-2.8)
Serum TSH 8.2 mU/L (<5) Which of the following statements would explain these results?
1) Abnormal thyroxine binding globulin
2) Amiodarone-induced hypothyroidism
3) 'sick euthyroid' syndrome
4) Spontaneous hypothyroidism
5) TSH secreting pituitary adenoma
Answer-2
The results show normal T4, low T3 with elevated TSH. These results are typical of amiodarone induced hypothyroidism which inhibits the peripheral conversion of T4
to T3.

50- A 65-year-old woman, a heavy smoker for many years, has had worsening dyspnoea for the past 5 years, without a significant cough. A chest X-ray shows increased lung size along with flattening of the diaphragms, consistent with emphysema. Over the next several years she develops worsening peripheral oedema. BP 115/70 mmHg. Which of the following cardiac findings is most likely to be present?
1) Constrictive pericarditis
2) Left ventricular aneurysm
3) Mitral valve stenosis
4) Non-bacterial thrombotic endocarditis
5) Right ventricular hypertrophy
Answer-5
This lady has Chronic Obstructive Airways disease and subsequent Cor Pulmonale leading to right heart failure. Non-bacterial thrombotic endocarditis is a condition
seen in frail ill individuals.

More Questions & Answers:-
Page1 Page2 Page3 Page4 Page5 Page6 Page7 Page8 Page9 
Page10

No comments:

Post a Comment